LSAT and Law School Admissions Forum

Get expert LSAT preparation and law school admissions advice from PowerScore Test Preparation.

 SMR
  • Posts: 16
  • Joined: Mar 21, 2014
|
#14462
Hi!

Thank you for your explanation. I see how answer choice A is correct. But I still don't see how the mechanistic approach was applied since all I know about the mechanistic approach is from the tips on page 4-28 and you stated that the argument didn't really apply those tips. So could you please explain the way the mechanistic approach is used in answer choice A and how "all or none" is a mechanistic way of approaching the argument? Maybe some kind of general definition for the mechanistic approach would help so I can see the bigger picture.

Also, you stated that the tips on page 4-28 were not really applied because the argument is not purely conditional. Does that mean that the mechanistic rules only apply to arguments that contain conditional reasoning?

Thank you again I appreciate it!
User avatar
 KelseyWoods
PowerScore Staff
  • PowerScore Staff
  • Posts: 1079
  • Joined: Jun 26, 2013
|
#14463
Hi SMR!

The mechanistic approach as described on page 4-28 is best applied to arguments where there is some sort of 'missing link.' In other words, there should be something in the premises not mentioned in the conclusion and something in the conclusion that is not in the premises. By linking those elements together, you can fill in the gap and justify the conclusion.

Finding a 'missing link' is often easiest and most helpful in arguments with conditional reasoning because the absolute nature of conditional statements mean that those links can end up proving your argument 100%. Consider the following example:

Premise: A :arrow: B

Conclusion: A :arrow: C

To justify that conclusion, I would take the element in the premise that is not in the conclusion (B) and the element in the conclusion that is not in the premise (C) and link them together:

Justify: B :arrow: C

If I combine that statement with my premise, my conclusion is now 100% proven:

Premise + Justify Statement: A :arrow: B :arrow: C
Conclusion: A :arrow: C

As you can see with question 8 on page 4-29, however, the missing link method will not always be the best way to think about your Justify question, especially when you don't have pure conditional reasoning. You can still take a 'mechanistic' approach to these questions by breaking down the stimulus into its component parts. But if there aren't elements in the premise not in the conclusion and vice versa, the missing link rules won't be as helpful.

So when I approach this question mechanistically, I break down the premise and the conclusion into their component parts, as Nikki already stated above. The premise is essentially that SOME mutations are random and the conclusion is essentially that ALL mutations are random. There's a logical leap between SOME and ALL that I need to bridge in my answer choice, but since this isn't pure conditional reasoning, it's not quite as clear cut as the example I provided above. But I still need to keep in mind that the correct answer choice is going to prove my argument 100% by getting me from SOME to ALL.

Technically, they could have given you an answer choice that might have been easier to spot and would have adhered more closely to the 'missing link' method (If SOME mutations are random, then ALL mutations are random). But sometimes the LSAT likes to make things a little hard on us so they get creative with the wording. By saying that it's an ALL or NONE situation, answer choice (A) essentially says "if SOME mutations are random, then ALL mutations are random." They just haven't said it in the way that you might have been looking for. But that's a pretty common occurrence on the LSAT so always make sure to read closely so you're clear on the meanings of the answer choices and the components of your stimulus :)

Hope that helps!

Best,
Kelsey
 SMR
  • Posts: 16
  • Joined: Mar 21, 2014
|
#14469
Thanks Kelsey! Your explanation helped a lot!
 avengingangel
  • Posts: 275
  • Joined: Jun 14, 2016
|
#28027
I almost chose D ... but I see why it's not right. But, to settle my wandering mind, would D be correct if, let's say, this was a necessary) assumption question, rather than a Justify question ?? Or could it be a correct answer to a Strengthen question ?? (If all the other answer choices were all different & wrong)
 Emily Haney-Caron
PowerScore Staff
  • PowerScore Staff
  • Posts: 577
  • Joined: Jan 12, 2012
|
#28043
Hi avengingangel,

D still wouldn't be a very good answer, even if this was an assumption question. We don't have to assume that environment has NO effect in order for the conclusion to be drawn; just that the environment won't have a sufficient effect to make mutation non-random. If it were a strengthen question, this could possibly (maybe) be a right answer, but would depend largely on the other answer choices.
 Tyler
  • Posts: 10
  • Joined: Jul 24, 2017
|
#37608
I chose answer choice c and my prephrase was "Something that proves all genetic mutation is random".

I just want to be clear why answer choice c is wrong and a is right. From what I understand, since the stimulus stated that since random mutations occurred within "some" of the population of bacteria, that means all genetic mutation is random.

I had my contenders down to A and C. I think I know why A is correct but I want to make sure. Since answer choice A confirms all genetic mutation is random if some bacteria have random genetic mutations, A is correct. Answer Choice C is conditional and doesn't prove the conclusion because of that aspect. Let me know what I am missing. Thanks!
User avatar
 Dave Killoran
PowerScore Staff
  • PowerScore Staff
  • Posts: 5853
  • Joined: Mar 25, 2011
|
#37623
Hey Tyler, just a quick question—were you able to read my explanation of this problem on the prior page? I went into detail about this problem there, and before answering here, I want to make sure just to avoid repeating myself.

As an aside, (C) isn't wrong solely because it's conditional, but because it's set up incorrectly. (A) is conditional, but it's correct, so it's not the mere presence of conditionality that is the issue.

Thanks!
 lsatnoobie
  • Posts: 52
  • Joined: Sep 18, 2017
|
#42275
The powerscore mechanistic approach tells us that “elements that are common to the conclusion and at least one premise normally do not appear in the correct answer.”

In this stimulus, genetic mutation appears both times. Can someone explain why E is not correct and More importantly, how doe one arrive at answer choice A mechanistically?
 lsatnoobie
  • Posts: 52
  • Joined: Sep 18, 2017
|
#42289
The following is from a previous explanation as to why C is wrong: The problem here is that the sufficient condition, "All random in bacteria," isn't know (just "some" is known, which while it could be "all," isn't known to be "all" for sure) so adding this statement to the premises doesn't give us "all random in life" as the conclusion.

This answer can be analogized as follows:

(C): If all high school football players are athletic, then football players at every level of the sport are athletic
Premise: Some high school football players are athletic.
Conclusion: None.”

I’m having trouble folllwing the football analogy. Can anyone help clarify? What do you mean by conclusion:None.

As for the sufficient condition “all random bacteria” not being known, how is this any different than Answer Choice A? Is it because Answer choice C is conditional by the word “IF” whereas Answer choice A is a statement?
 Adam Tyson
PowerScore Staff
  • PowerScore Staff
  • Posts: 5153
  • Joined: Apr 14, 2011
|
#42312
Hey again, noobie! What Dave meant in that response with the football analogy was that no conclusion could be drawn based on the premise (some h.s. football players are athletic) plus the answer choice (if all are, then all football payers at all levels are). The reason we cannot draw any conclusion there is because we don't know if ALL h.s. players are athletic - we only know that SOME are! That's the same issue as the one in the argument plus answer choice C: we cannot conclude anything based on the premise (some mutations were random - the ones in this bacteria in this one experiment) plus answer C (IF all bacterial mutations are random then all mutations everywhere are random) because we don't KNOW that all bacterial mutations are random. We only know that SOME are (these ones, from this experiment).

The correct answer justifies the conclusion because it adds the information that there are only two choices - all or none. We know from the premise that it is NOT "none", because we got some. If it isn't "none", and it must be either all or none, then it MUST be "all"! That was our conclusion, and this weird, counter-intuitive, incredibly unrealistic and very strong answer justifies it.

That's a common thread in many justify questions, btw - the answer choices tend to be very powerful, with extreme claims and extreme language. Hey, we are trying to PROVE something, not just strengthen it, so the more extreme the better, usually, right? The exception to that is when they ask which answer is REQUIRED to justify the conclusion. In that case, you want the answer that justifies the conclusion without going overboard, and an overly strong answer is probably incorrect. Watch out for those!

Let us know if that clears things up for you!

Get the most out of your LSAT Prep Plus subscription.

Analyze and track your performance with our Testing and Analytics Package.